You are on page 1of 10

CURRENT ELECTRICITY 3.

133

Solution. The value of Rshould be increased. As Problem 12. The current through a wire depends on
temperature of semiconductor increases, its tine as I = , + at, uhere I, =10 Aand u =4 As. Find
S
the decreases. As a result, the circuit resistance the charge that flous across a section of the wire in
resistivity tends to increase. To keep the 10 seconds.
decreases and current
constant, the value of Rhas to dq
reading of.ammeter (A) Solution. As I== , + af
beincreased. dt
Problem 10. Explain how electron mobil1ty cianges for +C
when (i) the tenperature of the conduc tor
q- (4 + at) dt=1,t + 2
agood conductor
decreased at constant potential difference und (in) ayplied At t=0, q =0, so constant of integration C =0.
is
potentaldifferenceis doubled at constnt temperature.
[CBSE OD 06C; D 17C]
9=ht +
2
Solution. Electron mobility of a conductor,.
4x(10)2 = 300 C.
et Att = 10s, q=10x 10 +
2
Problem 13. Auniform wine is cut inlo 10 segments
() When the temperature of the conductor decreases, inCTeasing in length in equal steps, the resistance of the
he relaxation timerof free electrons decreases, omonh is R and the resistances of the othr
so mobility u decreases.
segments increase in steps of 82. If the resistance of tle
() Mobility is independent of applied potential omgest segment is2 K,fnd the value of Rand hence find the
difference. resistance of the original wire.
Problem 11. Tuo wires X, Y hve the same resistivituy, Solution Resistance of first or shortest segment
tut their cross-sectional areas are in the ratio 2: 3 and = R2
lengths in the ratio 1:2. They are first connected in series Resistance of second segment
and then in parallel to a d.c. source. Find out the ratio of the
= R+8 =R+8x 12
drift specds of the electrons in the two wires for the twe cases.
[CBSE 08] Resistance of third segment
Solution. Current, I=enAv, =R+ 16= R+8 x 22
1 .:. Resistance of tenth segmernt
() In series, current I is constant. So, v, a
A =R+8x9= R + 72 2
v(X)_ Ay3=3:2 But resistance of longest segment =2 R
V() A, 2 2 R= R +72 R=72 Q
() In parallel, V = IR = constant Resistance of the original wire
1 = R+(R + 8)+(R +16)+..+(R +72)
R - 10 R +8(1 +2 +3+ ... +9)
But Ra = 10 R+ 8 x 45 -10 x 72 + 8x 45
A = 1080S.
Problem 14. The resistivity of the material of a
conductor of unifornit eross-section varis along its length as
p -p,(1 t ar) Deduce the expressin for the resistance of
or 2,2.4 the conductor, if its length is L. and area of croS-section is A.
y Ay ly 3 1 3 Solution. The resistance of the small length dr of the
enA,v (X)_ 2v,(X) conductor at distance x from its one end will be
Also,
enA, v,(Y) 3v,() =Po (1 + a)
dx
A
Hence, 2n(X) 4 The resistance of the length Lof the conductor will be
30,(Y) 3
D,(X) ==2:1.
2 (1+ x) dx
0
Problem 17. 'ahvbel: ta
odate
resistar
RSstuncY at

Solution Here
Problem 15. A crrent of 5 A s passng throuk a
RON-ner magestun wre ef cosssrton 04 m. A R, R 2 of R, 12 R,
ypoint the dirccta: of current density is at an angle ot !, 27°-300 K, a
60° wth the unt ector of ari ot c$ section. Find the 20x10x!
agritude ef clectric ficli at anv point of the conductor. R-R 12R-R,
(Resistivity of magnesium p - 44 10 2m) R,a
JEE Main July 21]
Solution t, =1000+ I, 1000 300
- 1300 Kor 1027C
Problem 18. Fie kkntx ceis cah ot nt restay
192 and nf 5V ae Onerted i TS arsl m palld ut
xternal rsistane RFr utat txuc of R.current msos
Nrallel cOmbinatits wil rean the same
IIEE Main Aug 21
I=|.A JAcos60° Solution.
J-2x5 103 2
Am
A 0.04 4

103
E= pl =44x 10x -Vm!
4 58
=11x 105Vm-1 R+5r

Problem 16. A carbon flament has a resistance of 100 2 I, =


at 0°C. What must be the resistance of a R+
copper filament
placed in series with carbon so that the combination has the
same resistance at all temperatures ? Temperature coefficient
of resistance of carbon = -0.0007 °C and that of
0.004°C1 copper is
Solution. As a =R-R
R!
.:. Change in resistance = a R, t
Let the resistance of copper placed in series
with
carbon at 0°C be R, so that the combination has the
same resistance at all temperatures. Then
Increase in resistance of copper per °C
-Decrease in resistance of carbon per °C
(a Ro t) for copper =(a R, t) for carbon 5
0.004 x Rxt=0.0007 × 100 x t R+5r
R+
5
0.0007 x 100 5R+r= R+5r
R= =17.52.
0.004
..R=r= 19.
CURRENT ELECTRICITY 3.135
Problem 19,Thelengths and radiii of three wires of same
ratios 2:3:4and 3:4:5 P'roblenm 21. Calculate the steady-state current through
are n the
paralleland
respectively. They 2 2resistor in the circuit shewn in Fig. 3.276. The nternal
tal included
joinedin currentin each wire.
in a circuit having 5 A resistance of the uF
battery is negligible and C=2 [CBSE
r Find [IIT] F 10)
Solution. Let R. Ry, R, be the resistances of the
2S2
wires.Then
3 4 A
&:Ry:Ry = 9 16 25 32

the currents in the three wires must be


The ratio of ratio.
4 S2
above
inverseof the
9 16 25 = 54:64:75 6V 2.8 2

=5 A, therefore
Ástotal current Fig. 3.276
5x 54 =1.40 A,
I, = 193 Solution. In the steady state, capacitor offers
infinite resistance to d.c., so no current flows through
5x 64
= 1.66 A, 4Q resistor, which is thus ineffective.
I, = 193
Effective resistance between points A and B is
5x75 - 1.94 A.
1, = 2 x3 =1.2 S2
193 R'=
2+3 5
Problem 20. Five 4 2 resistances, a 2 V batteru and a Total resistance of the circuit,
3.274. Find the
ater are coneted as shown in Fig. [BIT Ranchi 96] R= 1.2 +2.8 = 4.0 2
eeT reading.
42 Current drawn from the battery,
6
-=1.5 A
4Q R 4
P.D. between points A and B,
4Q V= IR' =15 x 1.2=1.8 V

42
2V A
.. Current through 2 2 resistor =
1.0-0.9 A.
variable rhenstat
42
wini
Problem 22. As shown in Fig 3.277, a
of 2k S2 is used to control the
500 S2 load.
potetial ditterene aTOSs
Fig. 3.274
shown in
Soution. The equivalent circuit is
Fig 3.275.
50 V
22 R
2V 500 2
42

22
Fiq. 3.2I}
potential
Bg. 3.215
resistance AB is 500 S2, what is the
(i) If the load ?
2 x2 +4 5Q difterence across the resistance
Equivalent resistance, R 2 +2 removed, what should be the [ISCE 96]
(i) lfthe load is betueen Band C?
E 2 =0.4 A. at BC to get 40 V
Ammeter reading l= R 5
PHYSICS-XI|
3136
When switch Sis closed, the parallel
Solution. Here R, =2 kQ- 2000 S2, R, =500 2, of all the three resistors is in the circuit.
RAR =500 2, Rc = R¡c -R,p =2000 - 500 =
1500 2
Total resistance of the parallel combination of RsC
resistance R' of the combination
11,1 13
is Coemqubiivnaalteurn
givenTheby
and R, R
1500 × 500
R'= 1500 + 500 -375 S2
or R'=
3
Total resistance of the circuit,
.. Ammeter reading,
R= Ran+ R'= 500+ 375 =875 2
0.3 r
Current in the circuit, =0.9 A.
R' r/3
50 V 2
A
R 875 2 35 Problem 24. If the
galvanometer in l2 :
() The potential drop across R, will bethe same as (Fig. 3.279) reads zero, find tlhe valueeof resistor theR.The(at
the potential drop across the parallel combination of source has negligible internal resistance If cool air is blo
across the wire wound resistance, what effect willbe
Rgc and R. and why ? notiooa
:. Potential drop across R,
-V-VAR2 10kQ
wire
= 50 - -x 500 =50 -28.57= 21.43 V. wound
35
12 V
(ii)If the load is removed, then the current will flow
through entire resistance RAc of the rheostat. 2V
50 V 1 Cell
.. Current, I' = A
2000 2 40
To obtain a potential drop of 40 V between Band C, Fig. 3.279
the required resistance BC must be Solution. As the galvanometer shows no deflection,
40 V so the p.d. across R is 2 V. If I is the current in the
= 1600S2. circuit, then
1/40 A
2
Problem 23. In the circuit shown in Fig. 3.278, both the IR = 2 volt or
R
ammeter and the cell have negligible resistance. Three
12
external resistors are identical. When the switch S is opened, But
the ammeter reads 0.6 A. What will the anmeter read when R+10
the switch S is closed ? 2 12
R R+10
On solving, R=2000 S2 = 2 kl.
When cool air is blown across the wire wOuu
in the
resistor, its resistance decreases. The current
circuit increases. The voltage across Rincreases. I
galvanometer will show some deflection.
Risshar
Fig. 3.278 Problemn 25. Aunifor1m wire of resistance
Solution. Let r be the resistance of each resistor. even. Fin
into a regular n sided polygon, where n is Corners oftk
When the switch Sis opened, the parallel combination equivalent resistance between (i) opposite
of upper two resistors is in the circuit. Their equivalent PoBygon and (i) adjacent corners of poBygon.
resistance is
Solution. Resistance of each side of polygoncorner.
R= OPposite
r+r 2 () Equivalent resistance between two pelyg
ofthe
polygon. The resistances of thetwo Parts
Potential drop across the cell, willbe
V= IR =0.6 x =0.3 r R R
2
R=R,2 2
CURRENT ELECTRICITY
a3.137
parts form aparallell combination. So the Problem 27. Space
Thesetwo
equivalent
resistance between two opposite corners of spheres of radii a und bbetween
(b> a) toe concentric
is flled with aconducting
medium of
polygon willbe resistivitu p. Proe that the resistance (R) betwen the two
the R R 1 1
R spheres wil! be
R'=
R 47\a (JEE Main April 19)
R + R, 4
2 Solution.
resistance betweeniwo adjacent corners
(i) Equivalent
The resistances of the two parts will be
polygon.
ofthe R, = Resistance of one side = R

R, = Resistance of remai ing (n -1) sides


connected in series
(n-1) R
Fig. 3.281
These two parts torm a parallel combi nation. So the dx
equivalent resistance of the polygon between two dR =p
be 42
adjacent cormers will
R (n-1) R
R* =1 (n-1) R a

R. (n-1)R
+

R=
Problem 26. Model ofa torch battery of length I is to be
Mmade up of a thin cylindrical bar of radius a' and a Problem 28. In a metre bridge, the wire of length 1 m
conQmtric thin cålindrical shell of radius b' filld in dR
betveen with an electrolyte of resistivityp(see Fig. 3.28 0a)) has a non-uniform cross-section such that, the variation dl
#the batteryis connected to a resistance of value R, sho0 of its resistance R with length Iis dR 1
that the maximum Joule heating in R will take place r Two equal
R:PIn resistances are connected as shown in Fig. 3.282. The
2d JEE Main Sept. 20} galoanometer has zero deflection when the jockey is at point
P. What is the length AP? JEE Main Jan 19]

dx R' R'

A
-1-1
ig. 3.280 (a) (b)
Fig. 3.282
Solution. Refer to Fig. 3.280(b). Maximum power is dR C
diSSipated in the external resistance when R =r. Solution.
2 R
R R=2C/
|r+ R
Resistance of the small element For balanced Wheatstone bridge,
dR P R 2C/i
2xd ’ 2/l =1
R' 22C(1-Vi)
l=-=0.25 m.
4
3.138 PHYSICS-X

Problem 29, Tuo cells of EMfs 1 V, 2 Vand internal As R, > R, these resistances in i
resstance 202 and 12 respectively are connected in
() series,fi)parallel. What should be the external resistance
RR r+ R, r+
increasing order
R + R,
n the crcuit o that the current through the resistance be the
same in the two cases ? In uwhich case more heat is generated The currents in decreasing order are :
n the cells ? (CBSE SP 08] 42 A, 1.4 A, 105 A, 0.6 A, 0.42 A
SoBution. Current in series circuit is given by -= 4.2
E, +¬, 1+2 3
,+,+ R 2+1+2 3+ R
=1.05 A
When the two cells are connected in parallel, r+ R

E,5 +E_1x1+2x2 5 =0.6 A


2+1 3 r+ Ry
1×2 2 =0.42 A
.4)
7t5 1l +2 3
=1.4 A
Current in the parallel circuit is given by r+
..5)
5 R +R
5
3 On dividing (1) by (2),
+ R 2 +3R
rt R 4.2
3
1.05
or 1
KË-4 or R=3r
As l, =1, On dividing (1)by (3),
3 5
3+ R 2 +3R r+ R 4.2
0.6
6+9R = 15 + 5R
1+
R=9/4 -2.25 2
More heat will be generated in series case due to Or
R=6r
larger resistance. From (1), E=4.2r
Problem 30. Acell of unknown emf E and internal
resistancer, tuo unknown resistances R, and R, ( R, > R,) Putting the above values in (4), we get
4.2r
and a perfect ammete are grven. The current in the circuit is =0,42
meQsured in fie different situations : r+3r+ 6r

() Without any external resistance in the circuit, r=12


(i) With resistance R, only, Hence E =4.2 V, r - 19, R, -32 and R, z01
(ii) With resistance R, only, Problemn 31, Abattery consists of 12 cells r
Some
(iv) With both R, and R, used in series combination, and each having anemf Eand interna! resistor r.
(v) With R, and R, used in parallel combination. cells in the battery are connected with wrong
battery is connected to another sOurce of emf2 3 4z
The current obtained in the five cases are: reauls
resistance 2 r. AH ammeter in the circuit 2Ainthe
0.42 A, 0.6 A, 1.05 A, 14 A, and 4.2 A battery and the SOurce aid each other and ho Y
but not necessarily in that order. ldentify the currents in the direction when they oppose each other. Find
five cases listed aboe and calculate E, r, R, and R. in the battery are connected with wrong polaritywrongr
ICBSE SP 08; D 12| connected
Soution. Total resistances in the five cases are : Solution. Suppose n cells aregiveforwardemt
the battery. Then (12 - n) cells
() r, (i) r + Ry (i) r + Rz cells give reverse emf.
.. Effective emf of the battery
(iv) r +R + R ()r+ =(12 -n)E-nE=(12 -2n)
CURRENT ELECTRICUY
Total rsistance of the circuit with battery and
cases
Decrease in heat production,
both
gren H- H H
12 r +2r=14r
H 100 =|1 H
100
Currentsin the two cases must be proportional to
l6 100 =36o.
thetwo cases
emtsin
the 25/
3
(12 -2n)E +2¬ Problem 34. In the circuit shoun in Fig. i,284, the heat
2
(12 -2n)E -28 produced in 552 resistor. due to the cunent flug throug
(14 -2n)8 3
it is 10 calorie per second. Find tle heat produced n 4S2
(10-2 n)E 2 resistor. {IITI
7-n 3 492 6 S2
5-n 2
of
14 -2n = 15 -3 n A 52

n=1
Fig. 3.284
one cell has beern connected with wrong polarity in
Le,
the battery. Solution. Heat produced per second in 5Q resistor,
Prnblem 32. In the ciTCuit shoan in ig. 3.283, cach P= 10cal s =10x 4.2 Js
and has an internal resistance of 0.2 2.
hatteryis of 5 V
IWat will be the reading of an ideal voltmeter connected As
R
= P
[IIT 97]
acrOssa battery ?
.:. Potential difference between points A and Bis
V=/PR =/10x 42 x 5 = /210 V
Current through 4 2 resistor,
V210 V210 A
4+6 10
Heat produced per second in 42 resistor
Fig. 3.283
V210 2
- R= x 4 Js1
Solution.Current in the circuit, 10
nE 8x5 210 4 cals=2 cal s,
|= = 25 A
8x0.2 100 4.2

Reading of the voltmeter, Problem 35. A heater is designed to operate with a


V=8-Ir=5 -25 x0.2 =5-5=0. power of 1000 W in a 100 V line. It is connecled in
Combination with a resistance of 10 2 and a resistance R to a
What should be the
Problem 33. Deternmine the percentage by which the 100 Vmains, as shown in Fig. 3.285.
lumination of a lanp willdecrease if the current drops by value of R sothat the heater operates with apower of 62.5 W?
[IIT 88)
20%.
Heater
Solution. If Ris the resistance of the lamp and I is 42

the current flowing for time t, then heat produced is R


H-'RI
When current drops by 20%, current in the circuit
100V

80% of L 80 Fig. 3.285


100
Heat produced, Solution. Resistance of heater,
y² 1002 = 102
H R=
1000
PHYSICS-XI

When the heater is connected as shown and the Problem 37. Detetmine te
power drops to 62.5 W, the p.d. across the heater 9iatörs'R,'R, id 'R, sAhoton tn Fig.curreMts
3.237.
would be
V=/PR =62.5x 10 =25 V
R R
.: P.D. across 102 resistor
= 100 -25 =75 V
Current in 10Q resistOr,
75 Pig.3.287
-=7.5 A
10
siiRÖn.Suppose currents l, and I, start from a
Current through heater, cells of emfs E, and , respectively, as
V 25
R' 10
= 2.5 A
Fig. 3.287(a). Then currents through the
R, R, and R, will be I, I, and I, +l, threeshown in
resistors
:. Current through resistance R
respectively.
- |-I'=75-2.5 =5 A Rg
P.D. across R = P.D. across heater = V'= 25 V

. Resistance, R= 25
=52.
5

Problem 36.Trettt tetors, vach ofW oin, Rq.3.287 (a)


coeted as shon in Fig. 3.286. A'bittv of2 iolt bnd of Applying
itinul rNtiOlbintis tötmetet airss he irttit. and 2, we get Kirchhoff
's second law to the loops 1

R 1 + R,(4 +1,)=¬,
or
(R +R,) I + Rl, =E, .(1)
R and
R
R, I, + R (l, + l,)=E,
(R, + Ry) 1, + R l, =E,
To eliminate I,, multiply (1) by (R, + R)and (2)
R and subtract. Then

(R +RyXR+ R,)I,-R;, -¬,(R,+ R,) -¬,8,


Sötieton. As three resistors of R ohm each are or
I, = E,(R, + R)-¬, R,
connected in parallel, their equivalent resistance R' is (R + R,)(R, + R,)- R;
given by
E(R, + R,)-¬, R
1.1. 1 3 + R R,+ R, R, + R,R
R R RR R
To eliminate I,, multiply (2) by (R, + R) and i"
R by R and subtract. Then
3
(R, +R)(R, +R) I, - RI, =E, (R, +R,)-A;*
Heat generated in the circuit is maximum when
Or
External resistance Internal resistance
(R, + R)(R, +R,)- R;
i.e., R=r
R
E, (R, +R,) -¬, R
or
3 R, R, +R, R, +R, R,
or R=3r =3x0.1 = 0.3 2. E R+¬, R
R R, + R, R +R, R
CURRENT ELECTRICITY
cells, eRsh af ennf e and
Problem 38.
it a chosed ciTcait so intesnal
that R,
iofacel is j9ined to the, negatie tesinaltheof, R
swum in Fig, 3.288{4).
3:2 Any, teopanis of fhe
y, an, external;resastAnge R. indtle
CHrNtinR

Fig, 3,289, (a)


Solution, The distribution of current for the given
network is shown in Fig. 3.290(6).
V,=6V
R
2092

ig. 3.288 (a) 1


V,= 10 V - 130 S2
Salution. Suppose m cells are connected on one
side of Rand remaining (n - m) cells on the other side
of R. Then the equivalent circuit will be of the form
shown in Fig. 3.288(b). The distribution of current is
shown. Fig, 3,289 (b)
n- m cells

Applying Kirchhoff's second law to the loop 1021,


y we get
101+ 201, =10 -6
m cells
101+ 201, =4 ...()
Similarly, from loop 0320, we get
D R 30(1-1,)-201, =6-5
Fig. 3.288 (b) 301 -501,=1 ...(i)
On solving equations (i) and (i), we get
Applying Kirchhoff 's second law to the loop
I=0.2 A
ABQCDPA, we get
(1- m)e =r(n-m) (r+ y)+ xR .(1) Thus the current flowing through the resistance R,
is 0.2 A.
Similarly, from the loop QEFPDCQ, we get Problem 40. Apart of the circuit n askeady skate along
me = mry - xk or mry = me + xR

or
me + xR resiskance, ete. is shon in Fig, 3,290 Cakylate th cergy
y=
mr
shorcd inn the capatkor. [IIT 86]

Substituting this value in (1), we get 1A


me + xR
(n-m)e=r(n- m) x+ mr
+ xR 32
32 52
D
On simplifying, we get 2A
x(mrn-rn + nR)=0 C#4uF 19
2A 19
x=0
B 22 E 4Q
Le., Current through R is zero. 3V
30
Problem 39. Fiud the curreut fivng throngh te
R R ot the cieuis sheeu in Fig. 3.289(4), G4 41A

Fig. 3.299
3.142
Problem 42. Fr the
the capacitor is
Solution. Inthe
there
steady
is no
state, when
current in the arm AB. Fig 3.292 poents and
tht l
otentieneter
tully charged,
the nk0anemtE. A
Applying Kirchhotf's first law at the junction A,
moved frmthe end Ato the end b.of thatobrted when
current in arm AD te
fauitspontenthtieoetciercrait
DE
tie detlection mthe galiatometer t
arm
=1+2 =3A =current in directern. What are the tue posible
junction B, could result in this obseroaton
Applying Kirchhoff's first law at the
current in arm EB
=2 -1=lA
Now Van =5x3 =15 V,
VpE =lx3 =3V,
VER 2x l=2 V
.:. P.D. across the ends Aand Bof the capacitor is Fiq. 3.292
V= Van + Vnr t VEp =15 +3 +2 =20 V
If the galvanometer deflection at the end Bis
Energy stored in the capacitor is (i) more (ii) less
u-cv²-x(4x 10)x (20) than that at the end A, which of the two faults, listed d
2 2 would be there in the circuit ? Give
answer in each case. reasons 1n Support of vou
=8x 10 . ICBSE OD m
Solution. Two possible faults for one-sidel
Problem 41. In the circuit shown in Fig. 3.291, find the deflection
current throu gh the 4 Q resistor. are as follows
[IIT 98]
32 A 2Q
(a) The positive terminals of all the cells are nt
B 22
connected to the point Aof the potentiometer,
1, (b) The emf of the driving cell is less than the emfa
9V the cell to be balanced.
82 82
()In case (a) the deflection at the end Bis more than
that at end A.
2S2 D
22 29 Reason. The two emfs support
Fig. 3.291
resultarnt emf becomes maximum ateach other and tik
the end B
(1) In case (b) the deflection at the end Bis less thar
Solution. Resistance between Band C on the right that at A.
hand side of the circuit = 8x 8 =4Q
8+8 Reason. The two emfs oppose each other andthe
resultant
Resistance between Aand Don the right handside maximum)deflection at the end B(where the main e
of the circuit 8x 8 =42
would be less than that at the end A
8+8 Problem 43. In the potentioneter circuit shoan
Fig. 3.293, the
Equivalent resistance of the circuit, balance
reason, where the balance (nuil) point Is at X. State
R=3+ 4 +2 =9Q p0int wil! be shifted wten
R
Current drawn from the battery,
9 V
I= =1A
92 X
B
At thejunction A the current of 1A is divided
eguallv between the 82resistance and the
Grcuit of resistance 8 2. remaining Fig. 3.293
At the junction B the current of 0.5 A is () Resistance R is increased, keeping all Parameters
equall between the 8SQ resistor and the divided
cirauit of resistance 8 2. remaining unchanged.
(ii) Resistance S is
Cucrent throughthe 4 2 resistor =0.25 A. (iii) Cell Pis replacedincreased,
by anotherkeeping Rconstant.
cell whoseemf islou
than that of cell Q ICBSEFa4;sP

You might also like